เรา (หมุนทฤษฎี) หมุนหลุมดำให้แรงจนมันจะแตกสลายด้วยแรงเหวี่ยงหรือไม่?


26

ฉันนึกภาพออกไม่ได้ว่ากองกำลังที่เกี่ยวข้องกับชีวิตของหลุมดำ ดังนั้นโปรดช่วยฉันค้นหาด้วยว่ามันเป็นไปได้หรือไม่ที่จะทำลายหลุมดำด้วยวิธีเฉพาะนี้


ฉันไม่แน่ใจ แต่ฉันคิดว่าเพื่อเพิ่มการหมุนอย่างมีประสิทธิภาพคุณต้องโยนสิ่งของลงไปและถ้าคุณทำคุณก็จะเพิ่มมวลมันด้วย ในขีด จำกัด ของสนามที่แข็งแกร่งของ GR การเพิ่มขึ้นของมวลที่สอดคล้องกันจะช่วยป้องกันแรงเหวี่ยงหนีจากกัน อย่างดีที่สุดมันจะกลายเป็นหลุมดำเคอร์ที่มี 'a' ใกล้กับ asymptotically 1 ถึงสำหรับหลุมดำแรงโน้มถ่วงจะชนะเสมอ! (ถ้าคุณต้องการแรงบิดมันขึ้นมาโดยไม่มีการขว้างปาสิ่งที่คุณจะต้องมีการออกไปค่อนข้างไกลและมันจะไม่เป็นที่มีประสิทธิภาพมาก)
คริส

คำตอบ:


23

พวกเรา (ตามหลักวิชา) สามารถหมุนหลุมดำให้แรงมากได้ไหมเพื่อที่มันจะถูกแยกออกจากกันด้วยแรงเหวี่ยง?

สำหรับเคอร์ - นิวแมน (การหมุน, แยก, แยก) หลุมดำของมวล , โมเมนตัมเชิงมุมJ , และชาร์จQ , พื้นที่ผิวของเส้นขอบเหตุการณ์ถูกกำหนดโดย A = 8 M [ M 2 + ( M 2 - a 2 - Q 2 ) 1 / 2 - Q 2 / 2 ] , ที่= J / M หลุมดำ extremalเกิดขึ้นเมื่อM 2 =MJQ

A=8M[M2+(M2a2Q2)1/2Q2/2],
a=J/M 2 ยิ่งไปกว่านั้นถ้าหลุมดำมีค่าใช้จ่ายมากเกินไปหรือคิดค่าใช้จ่ายมากเกินไปเป็นเวลาอวกาศ "เกินเหตุ" เคอร์ - นิวแมนซึ่งไม่น่าจะเป็นหลุมดำเลยจริง ๆ แต่เป็นเอกพจน์ที่เปลือยเปล่าM2=a2+Q2

ดังนั้นฉันตีความคำถามของคุณว่าถามว่าหลุมดำสามารถหมุนได้ถึงขีด จำกัด สูงสุดหรือเหนือกว่าเพื่อทำลายขอบฟ้าเหตุการณ์ มีความเป็นไปได้สูงมากที่ไม่สามารถทำได้

Wald ได้พิสูจน์ในปี 1974 ว่าเมื่อมีใครขว้างปาเข้าไปในหลุมดำเพื่อพยายามเพิ่มโมเมนตัมเชิงมุมของมันยิ่งใกล้กับหลุมดำมากเท่าไหร่มันยิ่งยากที่จะดำเนินการต่อกระบวนการนี้: หลุมดำที่หมุนเร็วจะผลักดันเรื่องนั้น จะเอามันเกินขีด จำกัด สูงสุด มีรูปแบบอื่น ๆ และแม้ว่าฉันจะไม่ได้ตระหนักถึงทั่วไปอย่างสมบูรณ์พิสูจน์ในทฤษฎีสัมพัทธภาพทั่วไปคลาสสิกความล้มเหลวอย่างต่อเนื่องของรูปแบบเช่นนี้เป็นแรงบันดาลใจอย่างดีจากการเชื่อมต่อระหว่างการเปลี่ยนแปลงของหลุมดำและอุณหพลศาสตร์

TH=κ/2π

κ=M2-a2-Q22M(M+M2-a2-Q2)-Q2

2

ฉันไม่มีคณิตศาสตร์ทั้งหมดที่อยู่ด้านบนของหัวของฉัน แต่จากความเข้าใจในแนวคิดของฉันมันเป็นไปไม่ได้

หลุมดำมีแรงดึงดูดมากพอที่แม้แสงจะไม่สามารถหลบหนีได้แม้จะอยู่ไกลจาก "พื้นผิว" (นั่นคือถ้าหลุมดำมีมวลต่ำพอที่จะยังคงมีพื้นผิวและไม่ยุบลงไปเป็นเอกฐาน) นั่นหมายความว่ามันจะต้องหมุนเร็วพอที่พื้นผิวเคลื่อนที่เร็วกว่าความเร็วแสงอย่างมีนัยสำคัญเพื่อให้มีโมเมนตัมเชิงเส้นเพียงพอ (มักเรียกว่า "แรงเหวี่ยง" ในกรอบอ้างอิงแบบวงกลม) เพื่อหนีซึ่งอ้างอิงจาก ทฤษฎีสัมพัทธภาพเป็นไปไม่ได้

การแผ่รังสีฮอว์คิงทำได้เพียงเพราะการแผ่รังสีแม่เหล็กไฟฟ้าเคลื่อนไปเกือบถึงมุมฉากกับ "พื้นผิว" ของหลุมดำและแสงสามารถโน้มตัว "งอ" ได้ด้วยแรงโน้มถ่วงเท่านั้นจึงไม่สามารถหยุดได้


2
คำตอบนี้มีปัญหาทางความคิดหลายประการ มันแสดงให้เห็นว่า "พื้นผิว" เป็นอย่างอื่นที่ไม่ใช่ขอบฟ้าเหตุการณ์ - แล้วมันคืออะไร? มันแสดงให้เห็นว่ารังสีฮอว์คิง จำกัด เพียงรังสีที่ไม่มีมวลเช่นเดียวกับแม่เหล็กไฟฟ้า - ไม่เป็นความจริง มันแสดงให้เห็นว่าหลุมดำไม่สามารถหยุดแสงได้ - ขอบฟ้าเป็นพื้นผิวที่สว่างเช่นนั้น
Stan Liou


0

เท่าที่เรารู้ไม่มีอะไรที่สามารถหยุดหลุมดำได้ สำหรับความคิดนี้จะทำให้ความรู้สึกครั้งแรกที่คุณจะต้องมองสิ่งที่เป็นที่รู้จักกันในปัจจุบันเกี่ยวกับหลุมดำ เมื่อคุณเข้าใจแล้วคุณจะเห็นว่าเนื่องจากความเข้าใจของเราในปัจจุบันเกี่ยวกับจักรวาลไม่มีอะไรที่เราสามารถทำได้เพื่อหลุมดำ

มันเป็นความจริงที่ Hawking Radiation สามารถส่งผลกระทบต่อหลุมดำ แต่นั่นเป็นเพียงสำหรับหลุมดำขนาดเล็กมากเท่านั้น

ในทางฟิสิกส์ไม่มีแรงเหวี่ยง - นี่เป็นความเข้าใจผิดที่หลายคนมี แต่มีแรงสู่ศูนย์กลาง

ป้อนคำอธิบายรูปภาพที่นี่


2
ภาระหน้าที่การโต้แย้งการเชื่อมโยงผ่าน xkcd: xkcd.com/123
Ilmari Karonen

3
@IlmariKaronen ขอบคุณสำหรับการลงคะแนนเสียงอย่างไรก็ตามคุณตระหนักดีว่าการ์ตูนไม่ได้พิสูจน์อะไรใช่มั้ย แรงของ Centripetal และแรงเหวี่ยงมีความแตกต่างกัน
FunctionR

2
เพื่อความยุติธรรมการ์ตูนชี้ให้เห็นว่าแรงเหวี่ยงเป็นแรงเฉื่อยซึ่งเป็นเรื่องจริง อาจเป็นเหตุผลที่สมเหตุสมผลที่จะตีความว่าแรงเฉื่อยนั้นเป็น "ไม่จริง" แต่ฉันไม่แน่ใจว่าสิ่งใดที่มีผลต่อคำถามนี้เนื่องจากแรงโน้มถ่วงเป็นแรงเฉื่อยเช่นกัน
Stan Liou

"คำตอบ" นี้ค่อนข้างคลุมเครือสร้างข้อความที่น่าสงสัยและไม่มีข้อสงสัยที่แท้จริง
Florin Andrei

0

น่าสนใจ กระบวนการนี้อาจส่งผลกระทบต่อการก่อตัวของหลุมดำในตอนแรก พิจารณาดาวหมุนที่ตายและเริ่มหดตัวเนื่องจากแรงโน้มถ่วง เมื่อมันหดตัวมวลของมันทั้งหมดจะถูกอัดเพิ่มขึ้นในรัศมีที่เล็กลง สิ่งนี้จะมีผลสืบเนื่องสองประการ: 1) แรงโน้มถ่วงที่ส่วนต่าง ๆ ของร่างกายจะเติบโตขึ้นด้วยการผกผันของรัศมีกำลังสองและ 2) ความเร็วในการหมุนจะเพิ่มขึ้นเนื่องจากการอนุรักษ์โมเมนตัมเชิงมุมและแรงขยายเนื่องจากการหมุน จะเติบโตขึ้นพร้อมกับอินเวอร์สของรัศมีคีบ ซึ่งหมายความว่ากำลังขยายตัวจะเติบโตเร็วกว่าที่ทำสัญญาและอย่างน้อยที่สุดในมุมมองของนิวตันกองกำลังขยายตัวจะเป็นผู้ชนะ จากมุมมองนี้ดูเหมือนว่าดาวฤกษ์ที่หมุนรอบจะไม่ก่อตัวเป็นหลุมดำ ...


คุณอาจชอบบทความนี้ slate.com/blogs/quora/2014/02/10/…
userLTK

-1

ลองทำสิ่งต่อไปนี้:

แบ่งกำลัง:

Fc=mv2RFg=GMmR2mv2/R=GMmR2v2=GMR

Rs=2GM/c2

v2=GM2GM/c2v=c2/2v=c2v=0,707c

0,707c

อย่างไรก็ตามเมื่อรัศมีขยายตัวการหมุนจะช้าลงโดยการอนุรักษ์โมเมนตัมเชิงมุม ... ดังนั้นฉันไม่คิดว่ามันจะแยกออกจากกัน ... อาจกลายเป็น "หลุมสีเทา" หรือไม่?

โปรดยกโทษให้ฉันด้วยถ้ามีข้อผิดพลาดพื้นฐานเกิดขึ้นฉันยังใหม่กับสิ่งเหล่านี้ ... : พี


3
นี่มันดูนิวตัน คุณพิจารณาความโค้งของกาลอวกาศใกล้กับหลุมดำหรือไม่? ดูเพิ่มเติมที่ "วิธีแก้ปัญหาเคอร์" สำหรับหลุมดำ
เจอรัลด์

ฉันชอบคำว่า "หลุมสีเทา"
userLTK

นี่เป็นคำตอบเก่า แต่ฉันคิดว่าฉันจะทิ้งมันไว้ที่นี่เพราะมันตอบโจทย์ความเร็วของหลุมดำ คุณต้องมีขอบทฤษฎีของหลุมดำที่ขอบฟ้าเหตุการณ์หมุนที่ C เพื่อหนีซึ่งเป็นไปไม่ได้แน่นอน physics.stackexchange.com/questions/207816/…
userLTK
โดยการใช้ไซต์ของเรา หมายความว่าคุณได้อ่านและทำความเข้าใจนโยบายคุกกี้และนโยบายความเป็นส่วนตัวของเราแล้ว
Licensed under cc by-sa 3.0 with attribution required.